Vektorraum Axiome beweisen

Neue Frage »

ThomasGabinski Auf diesen Beitrag antworten »
Vektorraum Axiome beweisen
Ich soll folgende Aufgabe lösen:

Ist


R - Vektorraum?

Das funktioniert ja eigentlich nach Schema f, indem ich die Axiome für Vektorräume auf die vorgegebenen Werte durchprobiere. Wenn sie gelten, handelt es sich um einen Vektorraum.

Ich weiss, dass x und y meine vorgegebnen Vektoren sind, doch was um himmels willen kann ich unter x=0 oder y=0 verstehen? Das irritiert mich leider sehr.
ThomasGabinski Auf diesen Beitrag antworten »

Tut mir leid ich meine ob folgendes ein Reeller-Vektorraum ist:



Mir geht es explizit um das oder. Was soll ich denn nun für x und y verwenden?^^
Guppi12 Auf diesen Beitrag antworten »

Deine Menge besteht aus Punkten des IR^2, in denen mindestens eine der Komponenten gleich 0 ist,
also zum Beispiel (0,0), (0,1), (pi, 0).

(1,1) liegt beispielsweise nicht drin.

Alternativ:


Hilft dir das?
ThomasGabinski Auf diesen Beitrag antworten »

Danke für den Tipp. Bedeutet das dann etwa, dass ich z.b. x als variable nutze welche ungleich null ist (ich gebe das vor) und y=0.

Hier ein paar angewendete Axiome:

1)x+0=0+x
2) lambda(x+0)=lambdax+lambda0

usw.
Guppi12 Auf diesen Beitrag antworten »

Du hast da glaub ich Grundsätzlich was falsch verstanden.

x und y sind in der Mengenschreibweise die Komponenten der Vektoren, die du untersuchen sollst, sie sind nicht die Vektoren selber.

Wenn du, wie du meintest x beliebig vorgibst und y gleich 0 setzt, wäre (x, 0) ein Element aus deiner Menge. Nimm dir ein weiteres, zb (z, 0) und rechne damit die Axiome nach.

Nebenbei ist deine Menge eine Teilmenge eines Vektorraums, nämlich IR^2 selber.
In diesem Fall genügt es, die Unterraumkriterien zu überprüfen.

Dafür muss deine Menge dann nämlich nur abgeschlossen bezüglich der Addition und skalarer Multiplikation sein. D.h. du nimmst dir zwei beliebige Vektoren deiner Menge, addierst sie und zeigst, dass die Summe dann wieder in der Menge liegt. Ebenso mit der Multiplikation eines Vektors mit einem Skalar.

Das funktioniert natürlich nur, wenn das ganze überhaupt ein Unterraum ist. Ist dem nicht so, sollte man sich ein Gegenbeispiel suchen, s.d. zum Beispiel das skalare Vielfache oder eine Summe von Vektoren aus der Menge nicht mehr in der Menge liegt. Das sollte man sich hier mal anschauen.
ThomasGabinski Auf diesen Beitrag antworten »

Danke zunächst einmal. Die Axiome an sich versteh ich prima. Auch wie man diese Überprüft. Ist ja an sich nicht schwer. Aber die Darstellungsweise bereitet mir gerade etwas Probleme. Bedeutet das R^2 nicht, dass es sich um ein zwei-Dimensionalen Raum handelt? Mich würde interessieren wie du auf zwei Mengen schließt?

Außerdem würde mich noch interessieren wie ich die Darstellung wähle zur Überprüfung der Axiome. Vektordarstellung ? also ,,Spaltendarstellung mit x oben und y unten? Tut mir leid für die ganzen fragen, aber ich möchte nur noch das heute schaffen und bin gerade leider etwas verwirrt wegen der Darstellung an sich halt.^^
 
 
Guppi12 Auf diesen Beitrag antworten »

Ja, für die Überprüfung der Axiome solltest du am besten
Zitat:
Spaltendarstellung mit x oben und y unten
wählen, das bietet sich hier an, da in der Definition direkt auf die beiden Komponenten Zugriff genommen wird.

Ich glaube ich habe gerade ein Problem damit, deine Frage zu verstehen.

Ist dir jetzt klar, wie die Menge

aussieht?

Ich denke mit den 3 Beispielen, was drin liegt und was nicht, sollte es eigentlich einigermaßen verständlich sein oder?

Du müsstest mir noch einmal genau sagen, wo deine Frage gerade liegt, bitte.
ThomasGabinski Auf diesen Beitrag antworten »

Also ich möchte nochmal erwähnen, dass ich dir sehr danke das du mir versuchst an einem späten Sonntag noch zu helfen.

Ich meine damit folgendes:

Gegeben sei die Menge

Nun soll ich hier die Axiome anwenden. Mich irritiert jedoch nur eine einzige Sache. Nämlich wie schreibe ich das (x,y) rechnerisch, für z.b. das Kommutativgesetz. Und mich würde dann noch interessieren, weshalb das so geschrieben wird, also welches Symbol das hier entscheidet. Muss ich z.b. in Spaltenform rechnen, da hier vom R^2 geredet wird?

Ich hoffe du verstehst was ich meine. Eventuell kannst du ja ein axiom einfach mal anwenden wenn du nicht verstehst was ich meine.

mfg
Guppi12 Auf diesen Beitrag antworten »

Ok, also ich überprüfe mal die Abgeschlossenheit bzgl. skalarer Multiplikation:

Sei also
und
Es gilt oder . Es folgt Aus folgt , ebenso für , also folgt oder , also

Wie gesagt, du machst es dir unnötig schwer, wenn du alle Vektorraumaxiome nachrechnest, es genügt die Unterraumkriterien nachzurechnen.
ThomasGabinski Auf diesen Beitrag antworten »

Super, vielen danke ich hab es verstanden. Jetzt interessiert mich nur noch eine Sache
wie kommst du darauf das (x,y) als Spalte dargestellt wird und keine Menge ist? Ich habe nämlich Angst, dass ich das mit einer Menge verwechseln würde. Kommt das eventuell vom , wobei der exponent die Anzahl der Zeilen angibt ? Dann wär das alles.^^
Guppi12 Auf diesen Beitrag antworten »

Mengen werden mit geschweiften Klammern geschrieben. Bei runden Klammern sind (bei 2 Komponenten) geordnete Paare(Paare sind 2-Tupel) gemeint. Dass die Elemente des Paars selbst keine Vektoren des IR^2 sein können ist aus dem Zusammenhang klar, denn dann wäre das Paar nicht aus dem IR^2 sondern aus dem (IR^2)^2.

Eine Menge bestehend aus 2 Vektoren des IR^2 wäre auch nicht Element des IR^2, sondern eine Teilmenge davon, das ist ein anderes Symbol.

D.h. macht garkeinen Sinn. Dann würde mann schreiben, wobei mit 0 dann die 0 des Vektorraums nicht die 0 der reellen Zahlen gemeint wäre.

Ob man die Elemente des IR^2 als Paar oder als Spalte schreibt ist einem selbst überlassen. Ich habe im Verlauf des Posts gemerkt, dass es wahrscheinlich genau das ist, was dich ein wenig verwirrt und bin deswegen zur Spaltenschreibweise übergegangen.
ThomasGabinski Auf diesen Beitrag antworten »

Es ist nämlich genau das, was mich aufgehalten hat die Aufgabe zu lösen. Also die Schreibweise der Komponenten des Vektors

Ich danke dir wirklich sehr für deine Hilfsbereitschaft so spät am Abend.^^
Guppi12 Auf diesen Beitrag antworten »

Habe noch ein klein wenig was ergänzt im edit, lies das nochmal. Vielleicht hilft dir das auch noch zum Verständnis.
ThomasGabinski Auf diesen Beitrag antworten »

Ja, das hilft etwas weiter. Kannst du mir kurz noch eine Frage beantworten?

Es handelt sich um fast dieselbe Menge. Der einzige unterschied ist, das nicht x=0 bzw. y=0 ist, sondern x+2y=0 ist.

Wie geh ich hier ran? Darf ich hier Äquivalentumformen? Sodass ich letztendlich x=-2y und y=-x/2 habe? Das setz ich dann beides in die Spaltendarstellweise von (x,y) ein, wobei folgendes herauskommt -> (-2y, -x/2). und hiermit überprüfe ich dann die jeweiligen wichtige Axiome?

Gerade ist mir noch eine Frage eingefallen. Bei z.b. dem Assoziativ gesetzt, brauche ich ja zwei Spalten. Eine ist (x,y) Was ist die andere? Wähle ich hier wieder (x,y) ?
ThomasGabinski Auf diesen Beitrag antworten »

Jemand eine Idee? Es ist ja nur (x,y) angegeben. Wie nutze ich diesen Vektor für z.b. das Assoziativgesetzt?
RavenOnJ Auf diesen Beitrag antworten »

Du musst doch einfach nur ein Gegenbeispiel angeben, um zu zeigen, dass das kein Vektorraum ist. In einem Vektorraum V muss beispielsweise gelten



Jetzt suche dir zwei geeignete Elemente aus deiner Menge und zeige, dass deren Summe nicht zu der Menge gehört.
ThomasGabinski Auf diesen Beitrag antworten »

Bedeutet das etwa, dass ich nun z.b. mit folgenden Vektoren die axiome anwenden darf, wobei bei beiden Vektoren x oder y immer 0 sein soll? Also z.b. v1=(-2,0) und v2=(67,0) ?

lg
RavenOnJ Auf diesen Beitrag antworten »

Die Axiome müssen für alle Elemente eines Vektorraums gelten. Findest du also Elemente der Menge, für die die Axiome nicht gelten, so hast du einen Widerspruch, d.h. es kann sich bei der Menge nicht um einen Vektorraum handeln. Die von dir genannten Elemente sind gerade kein geeignetes Beispiel.
ThomasGabinski Auf diesen Beitrag antworten »

Könntest du bitte in korrekter Form angeben, wie man das Assoziativgesetz mithilfe meiner vorgegebenen Eigenschaften aufschreibt? Mir geht es eigentlich viel mehr um die Schreibweise. Also ich habe ein Problem damit, wie man das nun aufschreibt. Für das Assoziativgesetzt gilt nun a + b = b +a

Wie verwende ich das aber nun auf mein Beispiel an? Ich habe doch nur einen Vektor angegeben oder? Also in diesem Fall wäre nun als Beispiel a=(x,0) und was ist b? Doch nicht auch b=(x,0). Die 0 habe ich gewählt weil ein Wert 0 sein soll.
RavenOnJ Auf diesen Beitrag antworten »

Du brauchst doch gar nicht so weit gehen und das Assoziativgesetz bemühen. Ein Vektorraum muss eine abelsche Gruppe sein und das ist hier schon verletzt.
ThomasGabinski Auf diesen Beitrag antworten »

Das mag sein. Aber mein Problem liegt ja nicht darin, die Axiome zu überprüfen, sondern viel mehr das vorgegebene rechnerisch darzustellen. Deshalb wollte ich gerne wissen wie denn nun das Assoziativgesetz für folgendes aussieht:


RavenOnJ Auf diesen Beitrag antworten »

Deine Aufgabe ist es, zu beweisen oder zu widerlegen, dass es sich bei der Menge um einen Vektorraum handelt. Dafür müssen alle Bedingungen und Axiome eines Vektorraums erfüllt sein, u.a. muss es sich um eine abelsche Gruppe handeln. Jetzt komm mal von deinem Assoziativgesetz runter, das ist erst mal vollkommen uninteressant. Weise zuerst nach, ob es sich um eine abelsche Gruppe handelt. Schon da wirst du sehen, dass dies nicht der Fall ist.
Captain Kirk Auf diesen Beitrag antworten »

Zitat:
Original von ThomasGabinski
Könntest du bitte in korrekter Form angeben, wie man das Assoziativgesetz mithilfe meiner vorgegebenen Eigenschaften aufschreibt? Mir geht es eigentlich viel mehr um die Schreibweise. Also ich habe ein Problem damit, wie man das nun aufschreibt. Für das Assoziativgesetzt gilt nun a + b = b +a

Das ist das Kommutativgesetz nicht das Assoziativgesetz.
Ihr redet grade beide grandios aneinander vorbei.

Nimm zwei Elemente deiner Menge a=(x,y), b=(x',y').
Eines mit erste Kooordinate Nulle, eines mit zweiter koordinat Null.
Was passiert furchtbares wenn man diese beiden addiert?
RavenOnJ Auf diesen Beitrag antworten »

@Captain Kirk
Da hätte er natürlich selber drauf kommen sollen.
ThomasGabinski Auf diesen Beitrag antworten »

Dann war es mein Fehler. Ich meine das Kommutativgesetz.^^

Daraus folgt dann:







Ist das so korrekt aufgeführt? Das wären nun Beispielzahlen aus meiner vorgegebenen Menge R^2. Ansonsten habe ich die Bedingung mit y=0 oder x=0 erfüllt. Wenn das korrekt sein sollte, würde mich noch interessieren, ob folgendes Auch erlaubt sei?

(0,6)+(0,-0) oder (0,0)+(0,0) oder (0,5)+(0,-33). Das sollten halt Beispiel sein zur Überprüfung der Axiome.

Damit ich die vorgegebene Menge auch nicht falsch interpretiere.^^
RavenOnJ Auf diesen Beitrag antworten »

Du scheinst nicht zu verstehen, worum es geht. Lies mal nach, welche Bedingungen für einen Vektorraum gelten müssen, z.B. hier. Geh mal bitte darauf ein, was ich vorher geschrieben habe und ignoriere das nicht beharrlich. Es ist vollkommen egal, ob das Kommutativ- oder Assoziativgesetz gilt, wenn schon die Grundvoraussetzung nicht stimmt, dass es sich nämlich bei der Menge um eine abelsche Gruppe handeln muss, damit ein Vektorraum vorliegen kann. Erst wenn klar ist, dass die Menge eine abelsche Gruppe ist, könntest du dazu übergehen, die anderen Axiome zu beweisen.
ThomasGabinski Auf diesen Beitrag antworten »

Ich weiss was die Axiome für eine abelsche Gruppe sind:

Assoziativgesetz, neutrales element vorhanden, inverses element vorhanden, Kommutativgesetz

Wenn ich das nun testen möchte. Also anschaulich, liegt mein Problem darin, die vorgegebene Menge zu übersetzten in eine rechnerische Schreibweise. Kann man etwa die vorgegebene Menge nicht rechnerisch darstellen? Tut mir leid, ich gebe wirklich mein bestes. Dennoch fällt es mir etwas schwer ihnen zu folgen.
RavenOnJ Auf diesen Beitrag antworten »

Zitat:
Dennoch fällt es mir etwas schwer ihnen zu folgen.

Hier ist es üblich, sich zu duzen.

Was du über abelsche Gruppen schreibst, ist richtig. Aber das grundsätzlichste, was in einer abelschen Gruppe G gelten muss, ist die Existenz einer Operation, die mit dem Symbol '+' oder Addition bezeichnet wird, sodass gilt:



Wenn schon diese grundsätzliche Voraussetzung nicht gilt, sollte es doch klar sein, dass keine abelsche Gruppe vorliegen kann.

Zu deinem konkreten Beispiel: Wenn du zwei Elemente deiner Menge betrachtest, und , diese addierst , dann kannst du schon sehen, dass diese Summe nicht in deiner Menge liegt. Die Grundvoraussetzung für eine abelsche Gruppe ist also nicht gegeben.
Amplitude Auf diesen Beitrag antworten »

Das bedeutet, das sofern bei meinen vorgegebenen Teilmengen von R^2 keine Operation ,,+" oder,,*" vorgegeben ist, ich diese nicht einmal überprüfen kann? Also nicht aus dem Bauch raus die Addition testen?
RavenOnJ Auf diesen Beitrag antworten »

Wenn keine Operation '+' vorgegeben ist, dann wird die übliche komponentenweise Addition stillschweigend angenommen.
Amplitude Auf diesen Beitrag antworten »

Was meinst du genau mit nicht der selben Menge? Also ich hab schon verstanden was du meinst, das ebend eine Verknüpung vorgegebener Mengen wieder in derselben Menge sein muss. Aber ist sie das hier nicht?
RavenOnJ Auf diesen Beitrag antworten »

Nein, das ist sie hier nicht. Lies dir noch mal die Spezifikation der Menge durch:



Also eine der Komponenten muss gleich 0 sein. liegt also nicht mehr in .
Amplitude Auf diesen Beitrag antworten »

Verdammt, jetzt verstehe ich das. Ich dachte bei der Eigenschaft bezieht sich das nur auf die Rechnung selbst und nicht auf das Ergebnis. Danke dir wirklich sehr für deine große Hilfsbereitschaft.

Wenn nun z.b. dieselben Angaben gemacht werden bis auf, dass x=0 ist und y=0 ist. Dann ist das auch kein Vektorraum oder? Solange nicht nur die Null als vorgegebene Menge gilt?
RavenOnJ Auf diesen Beitrag antworten »

Zitat:
Original von Amplitude
Wenn nun z.b. dieselben Angaben gemacht werden bis auf, dass x=0 ist und y=0 ist. Dann ist das auch kein Vektorraum oder? Solange nicht nur die Null als vorgegebene Menge gilt?


Das müsstest du jetzt mal etwas formaler fassen. Ich weiß nicht, was du für eine Menge meinst.
Amplitude Auf diesen Beitrag antworten »

Ich meine damit



Mir fällt gerade auf das es sich hierbei um einen Vektorraum handelt, da ich für jedes Element x und y zweier Vektoren oder mehr 0 wählen muss (Vorgegeben ist x=0 und y=0) Das Ergebnis ist immer ((0,0) also auch in der vorgegebenen Mengen x=0 und y=0

Ist das so richtig?
RavenOnJ Auf diesen Beitrag antworten »

Natürlich ist das ein Vektorraum, der trivialst mögliche, nämlich einfach nur der Nullpunkt, die Identität, die in jeder abelschen Gruppe vorhanden sein muss.

Du kannst auch spaßeshalber die Vektorraumaxiome überprüfen und wirst sehen, dass sie gelten.
Amplitude Auf diesen Beitrag antworten »

Danke ich fang es an zu verstehen. Jetzt scheint es aber wieder Berg ab zu gehen bei folgender vorgegebener Menge:

Ist die Menge der monoton steigenden Folge



ein R (Reeller)- Vektorraum?

ich weiss nicht mit was ich rechnen soll. Es ist doch explizit keine Folge angegeben die ich mit etwas Verknüpfen kann. Bitte nur Tipps und keine Lösung. smile
Amplitude Auf diesen Beitrag antworten »

Niemand eine Idee? Kann ich mir eventuell einfach eine monoton Steigende Folge ausdenken ?
Che Netzer Auf diesen Beitrag antworten »

Dass hier niemand antwortet, könnte daran liegen, dass du nun schon die dritte Aufgabe in diesen Thread stellst. Eröffne für neue Fragen neue Themen.
Neue Frage »
Antworten »



Verwandte Themen

Die Beliebtesten »
Die Größten »
Die Neuesten »